Themenbereiche Themenbereiche Profile Hilfe/Anleitungen Help    
Recent Posts Last 1|3|7 Days Suche Suche Tree Tree View  

vollst. Induktion

ZahlReich - Mathematik Hausaufgabenhilfe » ---- Archiv: Klassen 12/13 » Beweisführung » Archiviert bis 20. September 2002 Archiviert bis Seite 11 » vollst. Induktion « Zurück Vor »

Autor Beitrag
Seitenanfangvoriger Beitragnächster BeitragSeitenende Link zu diesem Beitrag

Christian (cube)
Suche alle Beiträge dieser Person in dieser Hauptrubrik
Neues Mitglied
Benutzername: cube

Nummer des Beitrags: 2
Registriert: 09-2002
Veröffentlicht am Mittwoch, den 18. September, 2002 - 09:35:   Beitrag drucken

Hey Leute
ich brauche noch mal eure Hilfe,
Ich soll folgenden Term mit hilfe der vollständigen Induktion beweisen (wenn der Term auch für n-1 stimmt, gilt er immer, oder ??)

n ist Element der Natürlichen Zahlen
x ist Element der positiven reelen Zahlen

x^n "größer-gleich" 1+n(x-1)

Aber wie mache ich das denn jetzt, wenn ich zwei unbekannte habe????

Wäre super wenn ihr mir wieder helfen könntet...
Bis dann Christian

Seitenanfangvoriger Beitragnächster BeitragSeitenende Link zu diesem Beitrag

Friedrich Laher (friedrichlaher)
Suche alle Beiträge dieser Person in dieser Hauptrubrik
Erfahrenes Mitglied
Benutzername: friedrichlaher

Nummer des Beitrags: 432
Registriert: 02-2002
Veröffentlicht am Mittwoch, den 18. September, 2002 - 12:47:   Beitrag drucken

Wenn bei Erhöhung des n auf n+1 der Zuwachs links >= Zuwachs rechts gilt ist die Ungl. bewiesen

xn+1-xn >= [ 1 + (n+1)(x-1) ] - [ 1 + n(x-1) ]

xn(x-1) >= x - 1 . Das stimmt ersichtlich für x >= 1

für y >= 1, 0 < x = 1/y <= 1, ergibt sich

(1/yn)(1/y -1) >= 1/y -1 ; da 1/y -1 < 0 wird bei Div. durch (1/y -1) das ">=" zu "<="

(1/yn) <= 1 was offensichtlich auch stimmt

womit x^n >= 1 + n(x-1) für alle Reellen Zahlen > 0 bewiesen ist.
Seitenanfangvoriger Beitragnächster BeitragSeitenende Link zu diesem Beitrag

Christian (cube)
Suche alle Beiträge dieser Person in dieser Hauptrubrik
Neues Mitglied
Benutzername: cube

Nummer des Beitrags: 3
Registriert: 09-2002
Veröffentlicht am Mittwoch, den 18. September, 2002 - 13:46:   Beitrag drucken

Danke erstmal für die Hilfe, aber ich hab dann noch einige Fragen/Verständnisprobleme:

-Kann ich dass auch mit n-1 beweisen?? Das will mein Lehrer nämlich von mir sehen...
wäre dann folgendes richtig:
x^n - x^(n-1) >= [1+n(x-1)] - [1+(n-1)(x-1)]


-Okay, wenn es nicht möglich ist, dann geht dass auch über n+1
so, dazu habe ich auch noch Fragen:
-die erste Gleichung ist klar.
-aber in dem moment wo y ins spiel kommt, verstehe ich es nicht mehr, was hat den y damit zu tun??

Viele Dank Christian
Seitenanfangvoriger Beitragnächster BeitragSeitenende Link zu diesem Beitrag

Friedrich Laher (friedrichlaher)
Suche alle Beiträge dieser Person in dieser Hauptrubrik
Erfahrenes Mitglied
Benutzername: friedrichlaher

Nummer des Beitrags: 437
Registriert: 02-2002
Veröffentlicht am Mittwoch, den 18. September, 2002 - 13:55:   Beitrag drucken

Substituiere in meiner Rechnung doch zu Beginn einfach

einfach n := m-1

und danach m := n
Seitenanfangvoriger Beitragnächster BeitragSeitenende Link zu diesem Beitrag

Friedrich Laher (friedrichlaher)
Suche alle Beiträge dieser Person in dieser Hauptrubrik
Erfahrenes Mitglied
Benutzername: friedrichlaher

Nummer des Beitrags: 438
Registriert: 02-2002
Veröffentlicht am Mittwoch, den 18. September, 2002 - 15:19:   Beitrag drucken

oder aber

x^(n-1)(x-1) >= x-1 ... kommt aufs gleiche hinaus

Beitrag verfassen
Das Senden ist in diesem Themengebiet nicht unterstützt. Kontaktieren Sie den Diskussions-Moderator für weitere Informationen.

ad

Administration Administration Abmelden Abmelden   Previous Page Previous Page Next Page Next Page